User Avatar
ajcarver1302
Joined
Apr 2025
Subscription
Free
PrepTests ·
PT102.S4.Q8
User Avatar
ajcarver1302
Monday, Jan 30 2017

Did you try to translate the without or no statements at the end of the stimulus? Did they make any sense? Please share.

PrepTests ·
PT104.S1.Q25
User Avatar
ajcarver1302
Monday, Jan 30 2017

In the video selecting B was primarily the result of process of elimination. I want an explanation that more positively supports B than JY's explanation in the video. In particular, and I know this might not always be possible, I want a more specific prediction of what I'm seeking in the answer choices before I confront them. Thoughts?

PrepTests ·
PT111.S1.Q2
User Avatar
ajcarver1302
Friday, Oct 28 2016

Is the word "whatever" in this case acting like a group 1 indicator? In answer choice A it seems like "whatever" is analogous to the group 1 indicator "whenever."

In general, when the word "what" is not used to ask a question, is it a group 1 indicator. For example, "What is a Jedi, is a force user . "

PrepTests ·
PT117.S4.Q7
User Avatar
ajcarver1302
Sunday, Jun 25 2017

I'm looking for a list of LR questions that use a necessary assumption question stem and for which the correct answer is a sufficient assumption. Can you tell me where to find one? Maybe you know the link to such a list on the 7sage forums or have been compiling such a list yourself?

User Avatar
ajcarver1302
Sunday, Jun 25 2017

7sage - Are these the question stems from actual questions? If so, could you share the Prep test, section number, and question number for each? I want to investigate a subset of these questions and I haven't easily found them using the LSAT question bank.

PrepTests ·
PT105.S2.Q25
User Avatar
ajcarver1302
Sunday, Apr 23 2017

Considering answer choices C and D, is it fair (or, better, backed up by other LSAT questions) to assume that we can interpret the phrases "It provides information that..." and "It presents a consideration that..." as effectively synonyms (by which I mean we should not be eliminating between answer choices that start with these phrases because of those phrases)?

PrepTests ·
PT103.S1.Q10
User Avatar
ajcarver1302
Sunday, Apr 23 2017

Are there other questions with the exact same question stem ("Which one of he following, if true, most seriously limits the effectiveness of adopting the argument's recommendation?")?

Or, are there other question stems starting with the phrase, "Which one of he following, if true, most seriously limits the effectiveness of..."?

PrepTests ·
PT104.S1.Q18
User Avatar
ajcarver1302
Saturday, Oct 22 2016

I did not think it was safe to assume that our inner ear would tell us we are moving if we were in a ship on rough waters (like JY does shortly after the 2:00 minute mark in the video explanation). I was suspicious that the LSAT was attempting to lure us into a trap answer by stating something reasonable in the real world (answer choice A) and having us assume that this real-world phenomena is related to the inner ear (which test takers might have no knowledge).

Thus, I eliminated answer choice A. I initially chose C. C was tempting, especially as the clock was ticking and with A gone, because it specifically mentions the inner ear.

On blind review I understood that C was wrong and that A was better.

If I had identified C as incorrect would have then likely been caught burning time because I would have been without an acceptable answer choice.

I know we're practicing to build intuition for questions like this one. Nevertheless, I wonder, three things:

1. Is an assumption being made as I describe in my first paragraph?

2. Do you have a rule to determine the degree of an assumption?

3. If an assumption has been made, do you have rules-of-thumb, or principles, concerning the appropriateness of making assumptions in answer choices that would a) continue to eliminate those answer choices that dupe us by playing on our knowledge of the real world while b) avoiding being eliminating answer choices where the assumption is acceptable.

After this problem, I think I have been too strict about eliminating answer choices that make assumptions. Since I was too strict, I eliminated A, and hence C seemed more tempting as I approached it. In the future I might "~" answer choice A and then compare A and C.

Links to lessons or other questions that highlight your points would be appreciated.

PrepTests ·
PT139.S4.Q19
User Avatar
ajcarver1302
Sunday, Dec 17 2017

Do you know of LSAT questions similiar to this one? By similiar I'm thinking of questions that (1) might tempt us to think the solution is mathematical when it is not or (2) is asking us to weaken an analogy and there are two seemingly close answer choices one which seems like it might be identifying a distinction with the internal reasoning of the argument and the other which finds another, perhaps broader, distinction between the two proposed analogous cases.

PrepTests ·
PT123.S3.Q20
User Avatar
ajcarver1302
Monday, Aug 15 2016

When considering the answer choices, J.Y. is looking for an argument that attacks a group with a different conclusion than the conclusion the author of answer choice would like to make.

He rejects answer choice B because it lacks such a group.

Q1. How else could we reject answer choice B?

I ask because I considered the answer choices by looking for an argument that supported its conclusion by assuming that the action in the conclusion is sufficiently significant to influence some expansive and important phenomena. To be precise, in the stimulus conclusion the train station should be demolish. The last sentence in the stimulus says "Preserving old buildings creates an impediment to new development, which is critical to economic health." I took the last sentence to be premise. (Q2. Why do you agree or disagree that the last sentence should be considered a premise?) This premise assumes preservation of old buildings to be sufficiently significant to harm long-term economic well-being. In answer choice B, the conclusion is document should be preserved. The reason is because losing or damaging even one document damages the integrity of the historical record (the expansive important phenomena like long-term economic well-being in the stimulus).

After writing this paragraph, I wonder, should I have applied the principle of assuming that the premises of the stimulus argument and answer choice arguments are true? In other words, above I identified a potential flaw (or weakness) in the premise (one thing will likely influence a much larger thing) but I should not seek flaws in premises. Instead, I should look for flaws between the premises and the conclusions they seek to support.

PrepTests ·
PT114.S1.Q9
User Avatar
ajcarver1302
Friday, May 12 2017

This question (question 9) and the next question are rated as difficult by 7sage (four and five circles respectively). If you took the entire section at once, did you skip them initially? If so, what triggered you to skip them? Do you have a process? An intuition?

PrepTests ·
PT105.S2.Q1
User Avatar
ajcarver1302
Saturday, Sep 03 2016

Why is this rated with two circles? It seems as easy or easier than many questions with one circle?

PrepTests ·
PT122.S2.Q9
User Avatar
ajcarver1302
Sunday, Sep 03 2017

How do you decide whether to check the other answer choices or to move on? In this case, would you select B and move on? Why or why not? Does the question type influence your decision?

PrepTests ·
PT112.S4.Q15
User Avatar
ajcarver1302
Thursday, Mar 02 2017

JY says, "...most questions that take on this phrasing, 'most helps to justify,' we'll learn later on is a specific subtype of strengthening question..."

What is the subtype?

In what lesson(s) is the subtype discussed?

Confirm action

Are you sure?